Đến nội dung

Hình ảnh

BĐT AM-GM


  • Please log in to reply
Chủ đề này có 339 trả lời

#301
Element hero Neos

Element hero Neos

    Trung úy

  • Thành viên
  • 943 Bài viết

$Cho \left\{x,y,z>0\begin{matrix} \\xyz=1 \end{matrix}\right. CMR:\frac{x^{2}}{x+y+y^{3}z}+\frac{y^{2}}{y+z+z^{3}x}+\frac{z^{2}}{x+z+x^{3}y} \geq 1$

đây mình trả lời rồi nè!



#302
NguyenPhuongQuynh

NguyenPhuongQuynh

    Hạ sĩ

  • Thành viên
  • 87 Bài viết

Cho x,y,z là 3 số dương thỏa mãn điều kiện $x^{2}+y^{2}+z^{2}=2012. Tìm giá trị nhỏ nhất của P =\frac{xy}{z}+\frac{yz}{x}+\frac{xz}{y}$



#303
Quoc Tuan Qbdh

Quoc Tuan Qbdh

    DragonBoy

  • Điều hành viên THCS
  • 1005 Bài viết

Cho x,y,z là 3 số dương thỏa mãn điều kiện $x^{2}+y^{2}+z^{2}=2012. Tìm giá trị nhỏ nhất của P =\frac{xy}{z}+\frac{yz}{x}+\frac{xz}{y}$

Ta có :

$P^{2}=\frac{x^{2}y^{2}}{z^{2}}+\frac{y^{2}z^{2}}{x^{2}}+\frac{z^{2}x^{2}}{y^{2}}+2(x^{2}+y^{2}+z^{2})$

Áp dụng BĐT $a^{2}+b^{2}+c^{2} \geq ab+bc+ca$ ( chứng minh bằng biến đổi tương đương )

Ta có ;
$P^{2} \geq 3(x^{2}+y^{2}+z^{2})=3.2012$

Suy ra $P \geq \sqrt{6036}=2\sqrt{1509}$

Dấu bằng khi $x=y=z=\frac{2\sqrt{1509}}{3}$


Bài viết đã được chỉnh sửa nội dung bởi Quoc Tuan Qbdh: 10-12-2015 - 19:38


#304
thanhtuoanh

thanhtuoanh

    Hạ sĩ

  • Thành viên
  • 54 Bài viết

Cho x,y,z là 3 số dương thỏa mãn điều kiện $x^{2}+y^{2}+z^{2}=2012. Tìm giá trị nhỏ nhất của P =\frac{xy}{z}+\frac{yz}{x}+\frac{xz}{y}$

Tổng quát:Cho x,y,z là 3 số dương thỏa mãn điều kiện $x^{2}+y^{2}+z^{2}=k$(k>0). Tìm giá trị nhỏ nhất của P =\frac{xy}{z}+\frac{yz}{x}+\frac{xz}{y}$



#305
Holutu

Holutu

    Binh nhất

  • Thành viên
  • 38 Bài viết

$(a+b+c)^{2}\geq 3(ab+bc+ca)$

giúp em chứng minh câu này với 



#306
Element hero Neos

Element hero Neos

    Trung úy

  • Thành viên
  • 943 Bài viết

$(a+b+c)^{2}\geq 3(ab+bc+ca)$

giúp em chứng minh câu này với 

            $(a+b+c)^{2}\geq 3(ab+bc+ca)$

Hay:     $a^{2}+b^{2}+c^{2}\geq ab+bc+ca$

Do đó:  $\frac{1}{2}((a-b)^{2}+(b-c)^{2}+(c-a)^{2}) \geq 0$


Bài viết đã được chỉnh sửa nội dung bởi Element hero Neos: 03-01-2016 - 08:20


#307
NguyenPhuongQuynh

NguyenPhuongQuynh

    Hạ sĩ

  • Thành viên
  • 87 Bài viết

Cho a>8.Tìm min$\sqrt{a^{2}+(\frac{a}{a-8})^{2}}$


Bài viết đã được chỉnh sửa nội dung bởi NguyenPhuongQuynh: 03-01-2016 - 09:12


#308
Element hero Neos

Element hero Neos

    Trung úy

  • Thành viên
  • 943 Bài viết

Cho a>8.Tìm min$\sqrt{a^{2}+(\frac{a}{a-8})^{2}}$

Bài này bạn PlanBbyFESN đã giải ở đây



#309
huynhtanduyan

huynhtanduyan

    Lính mới

  • Thành viên mới
  • 5 Bài viết

ai có tài liệu về bdt am-gm với  c.s ko ạ cho e xin



#310
cristianoronaldo

cristianoronaldo

    Thượng sĩ

  • Thành viên
  • 233 Bài viết

tiếp đây  $\sum \sqrt[3]{\frac{a^{2}+bc}{b^{2}+c^{2}}}\geq 9.\frac{\sqrt[3]{abc}}{a+b+c}$

Áp dụng bất đẳng thức AM-GM ta có:

$\sum \sqrt[3]{\frac{a^2+bc}{abc(b^2+c^2)}}= \sum \frac{a^2+bc}{\sqrt[3]{a(b^2+c^2).b(a^2+bc).c(a^2+bc)}}$

                                   $\geq \sum \frac{3(a^2+bc)}{a(b^2+c^2)+b(a^2+bc)+c(a^2+bc)}$

                                   $= \frac{3(\sum a^2+\sum ab)}{\sum ab(a+b)}$

Cuối cùng ta chỉ việc chứng minh:

$\frac{\sum a^2+\sum bc}{\sum ab(a+b)}\geq \frac{3}{\sum a}$

$\Leftrightarrow (\sum a)(\sum a^2+\sum bc)\geq 3\sum ab(a+b)$

$\Leftrightarrow \sum a^3+3abc\geq \sum ab(a+b)$

Đây chính là bất đẳng thức Schur với bậc 1

Dấu ''='' xảy ra khi a=b=c

$\Rightarrow$ Q.E.D


Bài viết đã được chỉnh sửa nội dung bởi cristianoronaldo: 31-05-2016 - 17:59

Nothing in your eyes


#311
anhminhnam

anhminhnam

    Trung sĩ

  • Thành viên
  • 155 Bài viết

Tổng quát:Cho x,y,z là 3 số dương thỏa mãn điều kiện $x^{2}+y^{2}+z^{2}=k$(k>0). Tìm giá trị nhỏ nhất của P =\frac{xy}{z}+\frac{yz}{x}+\frac{xz}{y}$

$P^2=\frac{x^2y^2}{z^2}+\frac{y^2z^2}{x^2}+\frac{z^2x^2}{y^2}+2(x^2+y^2+z^2)= \frac{1}{2}((\frac{x^2y^2}{z^2}+\frac{y^2z^2}{x^2})+(\frac{y^2z^2}{x^2}+\frac{z^2x^2}{y^2})+(\frac{x^2y^2}{z^2}+\frac{z^2x^2}{y^2}))+2k\geq y^2+z^2+x^2+2k=3k$

$P\geq \sqrt{3k}$

(chứng minh trên dùng bđt AM-GM (cauchy) )


:like Nếu bạn muốn đến nơi cao nhất, phải học cách bắt đầu từ nơi thấp nhất!  :like 

 


#312
anhminhnam

anhminhnam

    Trung sĩ

  • Thành viên
  • 155 Bài viết

$(a+b+c)^{2}\geq 3(ab+bc+ca)$

giúp em chứng minh câu này với 

Cái này thì @@, khá căn bản: 

$(a+b+c)^2\geq 3(ab+bc+ca)\Leftrightarrow a^2+b^2+c^2-ab-bc-ca\geq 0\Leftrightarrow \frac{1}{2}((a-b)^2+(b-c)^2+(c-a)^2)\geq 0$ (đúng)

vậy có đpcm


:like Nếu bạn muốn đến nơi cao nhất, phải học cách bắt đầu từ nơi thấp nhất!  :like 

 


#313
Oo Nguyen Hoang Nguyen oO

Oo Nguyen Hoang Nguyen oO

    Sĩ quan

  • Thành viên
  • 356 Bài viết

Cho $a,b$ là các số thực dương, chứng minh rằng: $\sqrt{\frac{a+2b}{a^2+2b^2}}+\sqrt{\frac{b+2a}{b^2+2a^2}}\leq \sqrt{\frac{8}{a+b}}$


Số hoàn hảo giống như người hoàn hảo, rất hiếm có.

Perfect numbers like perfect men, are very rare.

Rene Descartes

TỰ HÀO LÀ THÀNH VIÊN $\sqrt{MF}$

:icon6: :icon6: :icon6:


#314
tritanngo99

tritanngo99

    Đại úy

  • Điều hành viên THPT
  • 1644 Bài viết

Cho $x,y,z>0$ thỏa mãn: $\sum x^2y^2=3$. Tìm GTNN của biểu thức: $P=\sum \frac{1}{x}$

Mở rộng: Cho $x,y,z>0$. Chứng minh rằng:

$\frac{1}{x}+\frac{1}{y}+\frac{1}{z}\ge \frac{36}{9+x^2y^2+y^2z^2+z^2x^2}$


Bài viết đã được chỉnh sửa nội dung bởi tritanngo99: 22-06-2016 - 11:08


#315
lenadal

lenadal

    Trung sĩ

  • Thành viên
  • 161 Bài viết

Cho $x,y,z>0$ thỏa mãn: $\sum x^2y^2=3$. Tìm GTNN của biểu thức: $P=\sum \frac{1}{x}$

Mở rộng: Cho $x,y,z>0$. Chứng minh rằng:

$\frac{1}{x}+\frac{1}{y}+\frac{1}{z}\ge \frac{36}{9+x^2y^2+y^2z^2+z^2x^2}$

$+Ta có :$$\left ( \frac{1}{x}+\frac{1}{y}+\frac{1}{z} \right )^2\geq 3\left ( \frac{1}{xy}+\frac{1}{yz}+\frac{1}{zx} \right )\geq \frac{27}{xy+yz+zx}\geq \frac{27}{\sqrt{3\left ( x^2y^2+y^2z^2+z^2x^2 \right )}}=9$$
 
+Suy ra : $$\frac{1}{x}+\frac{1}{y}+\frac{1}{z}\geq 3$$

Bài viết đã được chỉnh sửa nội dung bởi lenadal: 23-06-2016 - 10:59

Lê Đình Văn LHP    :D  :D  :D 

http://diendantoanho...150899-lenadal/


#316
Hua Thi Mi Duyen

Hua Thi Mi Duyen

    Lính mới

  • Thành viên mới
  • 5 Bài viết

$\dpi{120} 0\leqslant x,y,z\leqslant 1,x\dotplus y\dotplus z\geqslant 2.C/Mxy\left ( x\dotplus 1 \right )\dotplus yz\left ( y\dotplus 1 \right )\dotplus zx\left ( z\dotplus 1 \right )$


Bài viết đã được chỉnh sửa nội dung bởi Hua Thi Mi Duyen: 01-07-2016 - 16:42


#317
Air Force

Air Force

    Trung sĩ

  • Thành viên
  • 145 Bài viết

*Hệ quả BĐT AM-GM(xét với các số không âm nhé)
1,$(a+b+c)^2\geq 3(ab+bc+ca)$
2,$a^3+b^3+c^3\geq 3abc$
3,$(ab+bc+ca)^2\geq 3abc(a+b+c)$
4,$\frac{1}{a}+\frac{1}{b}\geq \frac{4(=2^2)}{a+b}$ hay $\frac{1}{a+b}\leq \frac{1}{4}(\frac{1}{a}+\frac{1}{b})$ (tương tự với n số,bạn cũng có thể tổng quát lên dễ dàng )
*BĐT phụ :Không chỉ với AM-GM,BĐT phụ còn có tầm ảnh hương tới rất nhiều bài toán BĐT khác(đến nỗi mà đã có 1 topic riêng rất hay về phần nàyhttp://diendantoanho...-dẳng-thức-phụ/
mọi người tham khảo link trên,học thuộc dc thì càng tốt  :lol:
*Hệ quả BĐT C-S(CAUCHY-SCHWARZ )
1, 2 hệ quả rất quan trọng của BĐT C-S là (Schwars và Mincowski đều dc trình bày ở link mình đã đưa đàu bài )
*Bài toán định hướng
1,$\sqrt{\frac{a}{b+c}}\geq \frac{2a}{a+b+c}$
-Chứng minh
BĐT tương đương $\sqrt{a}(a+b+c)\geq 2a\sqrt{b+c}$ hay $a+(b+c)\geq 2\sqrt{a.(b+c)}$ (luuôn đúng theo AM-GM 2 số)
lời giải trên liệu có gì không tự nhiên không mọi người nhỉ 
(cái ta để tam ở đây là BĐT trên là 1 cách ta khử căn thức để làm BĐT đỡ "vất vả" hơn )
2,$\frac{a+b+c+d}{\sqrt[4]{abcd}}+\frac{16}{(a+b)(b+c)(c+d)(d+a)}\geq 5$
-Chứng minh :chú ý rằng $\frac{a+b+c+d}{\sqrt[4]{abcd}}=\frac{a+b}{2\sqrt[4]{abcd}}+\frac{b+c}{\sqrt[4]{abcd}}+\frac{c+d}{\sqrt[4]{abcd}}+\frac{d+a}{\sqrt[4]{abcd}}$
đến đây, nếu trở ại bài toán,ta chỉ cần áp dụng BĐT AM-GM cho 5 số là xong
(cái mình muốn nói ở đây là viêc tách hạng tử nhiều bạn chắc chắn sẽ thắc mắc tại sao lại tách như trên .Trong tầm kiến thức ở AM-GM(tức là ta chỉ xem xét các khả năng giải bằng AM-GM cho bài toán trên ),mình xin giải thích như sau
+,nếu áp dụng trực tiếp AM-GM ,chắc chắn sẽ không đem lại kết quả rồi
+,chú ý rằng bài toán trên không hề cho đk ẩn(ngoài không âm)nên ta khả dĩ không thể áp dụng AM-GM ĐỂ RỒI ĐƯA VẾ TRÁI VỀ 1 BIỂU THỨC CÒN ẨN
+,từ đánh giá thứ 2,ta buộc phải tìm cách triệt tiêu hoàn toàn tất cả ẩn sau khi đã áp dụng AM-GM
tức là phải tách 1 trong 2 hạnh tử trên ra thành tổng của 1 số hạng tử náo đó
+,Phân thức $\frac{16abcd}{(a+b)(b+c)(c+d)(d+a)}$ tuyệt vời này chắc cũng chẳng ai tìm cách tách nó,vì vậy ta tìm cách tách phân thức $\frac{a+b+c+d}{\sqrt[4]{abcd}}$
+'dựa vào phân thức cố định trên,ta phải tách phân thức $\frac{a+b+c+d}{\sqrt[4]{abcd}}$ thành tổng các phân thức có mẫu là $k(a+b),k(b+c)..$ để triệt tiêu mẫu(số pphân thức tách được là 4 )
+,chọn k= 1(vì sao?chẳng vì sao cả,nếu không dc ta lại chọn số khác ,cho dễ tihs thôi) ta dc 1 dạng $\frac{a+b}{x}+\frac{b+c}{y}+\frac{c+d}{z}+\frac{d+a}{t}$
+,chọn x=y=z=t ,ta đc tổng trên bằng $\frac{2(a+b+c+d)}{x}$ vậy hiển nhiên $x=2\sqrt[4]{abcd}$
đến đây xem như ta đã hoàn thành trọn vẹn bài toán 
3,$(a+b+c)(ab+bc+ca)\leq \frac{8}{9}(a+b)(b+c)(c+a)$
(mình xin nêu khái quát về phép đổi biến p.q.r,nếu có dk mình sẽ giới thiệu ở BĐT schur, hoặc kĩ thuật đặt ẩn phụ )
để giảm bớt khối lượng tính toán,trong trường hợp đủ bộ,ta có thể đặt 
$a+b+c=p;ab+bc+ca=q;abc=r$,khi đó hiển nhiên là $p\geq 3r;q\geq 3p;p^2\geq 3q,q^2\geq 3pr$
(còn rất nhiều BĐT nữa mình sẽ giới thêu sau và cũng khuyến khích mọi người thử xây dựng)
*1 hằng đẳng thức quý vô bờ bến mà mọi người cần nhớ là $(a+b)(b+c)(c+a)= (a+b+c)(ab+bc+ca)-abc$
chứng minh thuần bằng đại số
trở lại bài toán ,BĐT viết lại như sau $9r\leq pq$ và điều này là hiển nhiên 
4,$\frac{a^2}{b+c}+\frac{b^2}{c+a}+\frac{c^2}{a+b}\geq \frac{a+b+c}{2}$
Mặc dù BĐT có thể giải hết sức đơn giản bằng C-S tuy nhiên cái mình muốn nói ở đây(và cũng là cái mình yếu nhất)là khử mẫu-điểm rơi
+,trước tiên ta để ý là không thể cộng trực tiếp các số hạng ở VT lại,làm như vậy quá "mạo hiểm"  :excl: 
+,tư tưởng của ta là khử mẫu cho dễ tính toán và AM-GM có thể làm điều này giúp bạn 
$\frac{a^2}{b+c}+\frac{b+c}{x}\geq 2\sqrt{\frac{a^2.(b+c)}{x.(b+c)}}= 2\frac{a}{\sqrt{x}}$
cái quan trọng giờ là con x ấy là con lào ?ta không dc chọn mò nó,vì sao?vì theo AM-GM thì các số hạng được vinh dự tham gia áp dụng phải bằng nhau thì mới có đẳng thức ,đây cũng là mấu chốt để tìm x,
+,ta phải có $\frac{a^2}{b+c}=\frac{b+c}{x}$,(1)chú ý hơn 1 tí nữa,trong hầu hết các BĐT sử dụng AM-GM hay C-S(ta xét 3 biến a,b,c) đẳng thức xảy ra khi a=b=c hoặc (a=b,c=0)và các hoán vị,trong bài toán này nếu thay a=b=c,ta sẽ có đẳng thức(2)
từ (1) và(2) ta đễ tìm dc x,phần còn lại bạn trình bày nốt nhé 



#318
Sherlock Homes

Sherlock Homes

    Binh nhì

  • Thành viên
  • 13 Bài viết

Ai rảnh thì giải giúp nhé

Chứng minh rằng với mọi a,b,c>0 thì

 $\frac{a}{b}+\sqrt{\frac{b}{c}}+\sqrt[3]{\frac{c}{a}}>\frac{5}{2}$


Bài viết đã được chỉnh sửa nội dung bởi Sherlock Homes: 20-08-2016 - 10:26

Impossible is nothing --- ADDIDAS ---

     

                                                                                BELIVEVE THAT --- Roman Reigns ---


#319
tritanngo99

tritanngo99

    Đại úy

  • Điều hành viên THPT
  • 1644 Bài viết

Ai rảnh thì giải giúp nhé

Chứng minh rằng với mọi a,b,c>0 thì

 $\frac{a}{b}+\sqrt{\frac{b}{c}}+\sqrt[3]{\frac{c}{a}}>\frac{5}{2}$

Ta có: $\frac{a}{b}+\sqrt{\frac{b}{c}}+\sqrt[3]{\frac{c}{a}}=\frac{a}{b}+\sqrt{\frac{b}{4c}}+\sqrt{\frac{b}{4c}}+\sqrt[3]{\frac{c}{27a}}+\sqrt[3]{\frac{c}{27a}}+\sqrt[3]{\frac{c}{27a}}\ge 6\sqrt[6]{(\frac{1}{2})^2*(\frac{1}{3})^3}>\frac{5}{2}\implies Q.E.D$



#320
hoangpro1811

hoangpro1811

    Lính mới

  • Thành viên
  • 8 Bài viết

Cho $a ,b,c$ là các số dương và $a.b.c=1$ .Tim max của $\sum \frac{a}{a+b^2+c^2}$


Bài viết đã được chỉnh sửa nội dung bởi tpdtthltvp: 31-08-2016 - 16:51





1 người đang xem chủ đề

0 thành viên, 1 khách, 0 thành viên ẩn danh